LSAT and Law School Admissions Forum

Get expert LSAT preparation and law school admissions advice from PowerScore Test Preparation.

 Administrator
PowerScore Staff
  • PowerScore Staff
  • Posts: 8916
  • Joined: Feb 02, 2011
|
#47408
Complete Question Explanation
(The complete setup for this game can be found here: lsat/viewtopic.php?t=17028)

The correct answer choice is (E)

The question stem establishes that T testifies first:
D10_game #4_#20_diagram 1.png
With T testifying first, U must testify immediately after S in order to conform to the first rule:
D10_game #4_#20_diagram 2.png
Of course, from the second rule, U must also testify earlier than R and W:
D10_game #4_#20_diagram 3.png
With this sequence in place, only two basic scenarios are possible, each based on complying with the third rule:
D10_game #4_#20_diagram 4.png
Accordingly, answer choice (E) is correct.
You do not have the required permissions to view the files attached to this post.
User avatar
 andrewlee
  • Posts: 2
  • Joined: Sep 25, 2022
|
#103301
I don't know if I'm missing something obvious, but doesn't C say "Sanderson testifies third," and isn't that possible in the TM version? How is C not correct as well?
 Adam Tyson
PowerScore Staff
  • PowerScore Staff
  • Posts: 5153
  • Joined: Apr 14, 2011
|
#103313
You've misread answer C, Andrew; It says Sanderson testifies FOURTH!

Read carefully!

Get the most out of your LSAT Prep Plus subscription.

Analyze and track your performance with our Testing and Analytics Package.